1. Which 3 numbers have the same answer whether they’re added or multiplied together?
^Brainliest anyone^

Answers

Answer 1

Answer:

1, 2, 3

Step-by-step explanation:

1 + 2 + 3 = 6

1 * 2 * 3 = 6


Related Questions

Find the value of x.

Answers

Answer:

cos 62=adjacent/hypotenuse

cos62=x/6

x=cos 62*6

x=2.8units

Answer:

x ≈ 2.8 units

Step-by-step explanation:

find measure of ∠B:  180-(90+62) = 28°

law of sines:

sin90°/6 = sin28°/x

1/6 = sin28°/ x

cross-multiply to get:

x = 6·sin28°

x ≈ 2.8 units

Help with this question, not sure what the answer is

Answers

The answer is B, you know the zeros or solutions are positive 3 and 7 means no that they will be negative in the equation. And the parabola isn’t stretched or compressed so it doesn’t have a leading coefficient. Hope that helps!

Answer:

3rd option

Step-by-step explanation:

The zeros ( where the graph crosses the x- axis ) are x = 3 and x = 7

The corresponding factors are then (x - 3) and (x - 7)

The equation is then

f(x) = a(x - 3)(x - 7) ← a is a multiplier

To find a substitute any point on the graph into the equation.

Using (8, 15 ) , then

15 = a(8 - 3)(8 - 7) = a(5)(1) = 5a ( divide both sides by 5 )

3 = a

Thus

f(x) = 3(x - 3)(x - 7)

HELPPP ME PLEASE WILL MARK BRAINLIEST IF YOU GIVE ME THE CORRECT ANSWER

Answers

Answer:

Option (3)

Step-by-step explanation:

Properties for the vertical stretch or shrink of a function,

If a function is f(x) = k|x|

1). Function will be vertically stretched if k > 1

2). Function will have a vertical shrink if 0 < k < 1

To fit the given data graph of f(x) = |x| will vertically shrink without any translation along x or y-axis.

And the value of k will be, 0 < k < 1.

Therefore, g(x) = [tex]\frac{1}{3}f(x)[/tex] will be the transformed graph.

Therefore, Option (3) will be the correct option.

Find the average rate of change of f ( x ) = 3 x 2 − 9 on the interval [ 2 , b ] . Your answer will be an expression involving b

Answers

Answer:

[tex]m = 3b+6[/tex]

Step-by-step explanation:

Given

[tex]f(x)=3x^2 - 9[/tex]

Required

The average rate over [tex][2,b][/tex]

Average rate (m) is calculated using:

[tex]m = \frac{f(b) - f(a)}{b - a}[/tex]

Where

[tex][a,b] = [2,b][/tex]

So, we have:

[tex]m = \frac{f(b) - f(2)}{b - 2}[/tex]

Calculate f(b) and f(2)

[tex]f(x)=3x^2 - 9[/tex]

[tex]f(b)=3b^2 - 9[/tex]

[tex]f(2)=3*2^2 - 9 = 12 - 9 = 3[/tex]

So, we have:

[tex]m = \frac{f(b) - f(2)}{b - 2}[/tex]

[tex]m = \frac{3b^2 - 9 - 3}{b - 2}[/tex]

[tex]m = \frac{3b^2 - 12}{b - 2}[/tex]

Expand the numerator

[tex]m = \frac{3b^2 + 6b-6b-12}{b - 2}[/tex]

Factorize

[tex]m = \frac{b(3b + 6)-2(3b+6)}{b - 2}[/tex]

Factor out 3b + 6

[tex]m = \frac{(b -2)(3b+6)}{b - 2}[/tex]

Cancel out b - 2

[tex]m = 3b+6[/tex]

Anthony bought a new rectangular table for his living room the length of the table is 4 feet and the perimeter of the table is 14 feet what is the width of the table ?

A. 4 feet

B.5 feet

C. 2 feet

D.3 feet

Answers

Answer:

Option d

Step-by-step explanation:

Length = 4 ftPerimeter = 14 ftWidth = ?

→ Perimeter of rectangle = 2(l + w)

→ 14 = 2(4 + w)

→ 14 ÷ 2 = 4 + w

→ 7 = 4 + w

→ 7 - 4 = w

3 ft = w

Therefore, width is 3 ft. (Option D)

3 feet so it’s ddddddd

Classify the polynomial 8x^3 +6x^2 -3x +1 according to its degree and number of terms.?

Plz fast

Answers

Answer:

Find the degree, leading term, and leading coefficient.

Polynomial Degree:  

3

Leading Term:  

8x^3

Leading Coefficient:  

8

Step-by-step explanation:

8 is the correct ans

43. If L is the midpoint of KN an MP, which methods can be used to prove the triangles are congruent

Answers

Answer:

Step-by-step explanation:

Two triangles are said to be congruent if all their corresponding sides and corresponding angles are equal to each other.

Given that L is the midpoint of KN an MP, hence:

KL = LN; and ML = LN

Also, ∠KLM = ∠PLN (vertical opposite angles are congruent to each other)

Since ∠KLM = ∠PLN, KL = LN; and ML = LN, we can hence say that triangle KLM and triangle LNP are congruent triangles using Side-angle-side (SAS) triangle congruency theorem.

The SAS theorem states that if two sides and one included angle of one triangle is equal to two sides and an included angle of another triangle, then both triangle are congruent

Write equivalent fraction for the given number : 2/4 = _______ , ____________ , ______________, _ _________

Answers

Answer:

1/2, 4/8, 5/10, 6/12 (examples of equivalent fractions)

Step-by-step explanation:

all you have to do is keep the ratio the same, so whatever change you make to one side of the fraction, you do the same to the other

here's an example: 2/4 divided by 2/2 = 1/2

- you would divide 2 by 2 AND 4 by 2

1/2 , 3/6 , 4/8 , 5/10

angle H and angle D are supplementary. If angle H has a measure of 128 degrees, find the measure of angle D

Answers

m<H + m< D = 180°

Since m<H and m<D are supplementary angles.

128°+m<D =180°

m<D = 180-128

m<D = 52°

Answer: m<D = 52°

Answer:

angle D=52 degree

Step-by-step explanation:

In supplementrary angles sum of two angles is always 180 degree

angle H +angle D=180 degree (since they are supplementary angles)

sbstitute the value of H

128 + angle D=180

angle D=180-128

angle D=52 degree

therefore angle D is 52 degree.

What is the area of the shaded region? 2 cm 2cm
4cm 4cm

Answers

Answer:

12?

Step-by-step explanation:

4x4= 16(area)

2x2=4(area menor)

16-4= 12

divide (6x³+5x²-4x+3) by (2x+3)​

Answers

Answer: 3x^2-2x+1

Step-by-step explanation:

Answer:

3x² - 2x + 1

Step-by-step explanation:

To get 3x²

6x³ ÷ 2x

To get -2x

-4x² ÷ 2x

To get +1

2x ÷ 2x

What is the measure of ADB?

Answers

The answer is A 109

Explanation:
Solve for x when doing 6x+19=7x+4
Then plug in the value of x into 6x+19
6x+19=7x+4
-4 each side
6x+15=7x
-6x from each side
15=1x
15=x
Then plug it back in:
(6 [15] +19)
(90+19)
109

The answer is A: 109

Please help if you can!!!!

Answers

Answer:

about 22 square units

Step-by-step explanation:

sana makatulong

Answer:

C: About 22 square units

Step-by-step explanation:

YW

The addition Of 5 Times A Number X and 17 ?

Answers

Answer:

5x+17

Step-by-step explanation:

5 times a number, X, can be represented as 5x. If we want to add 5x and 17, we can say 5x+17.

Sophia determined that the expression Negative (4 x minus 5) + 2 (x minus 3) is equivalent to –2x – 5 by using the steps below.


Expression: Negative (4 x minus 5) + 2 (x minus 3)
Expression: –2x – 5
Step 1:
Choose a value of x
Choose x = 1.
Choose x = negative 1.
Step 2: Substitute
Negative (4 (1) minus 5) + 2 (1 minus 3)
Negative 2 (negative 1) minus 5
Step 3: Evaluate
Negative (4 (1) minus 5) + 2 (1 minus 3) = negative (negative 1) + 2 (negative 2) = negative 3
Negative 2 (negative 1) minus 5 = 2 minus 5 = negative 3
Step 4:
Compare
Since –3 = –3, the two expressions are equivalent.

Which explains whether Sophia is correct?

Answers

Answer:Hope This Helps ☺️

Step-by-step explanation:

She is not correct because she did not substitute the same number in both expressions in Step 1

Step-by-step explanation:

CASE 1: substitute 1 for x to both sides of the equations

L.H.S

-(4x-5)+2(x-3)

-(4 (1) - 5)+ 2(1-3) = - (-1) + 2(-2) = 1 - 4 = -3

R.H.S

-2x - 5

-2(1) - 5 = -2-5 = -7

Hence for x= 1

-(4x-5)+2(x-3) ≠ -2x -5

Because -3 ≠ -7

CASE 2: substitute -1 for x to both sides of the equations

L.H.S

-(4x-5)+2(x-3)

-(4 (-1) - 5)+ 2(-1-3) = - (-9) + 2(-4) = 9 - 8 = 1

R.H.S

-2x - 5

-2(-1) - 5 = 2-5 = -3

Hence for x= -1

-(4x-5)+2(x-3) ≠ -2x -5

Because 1 ≠ -3

Answer:

She is not correct because she did not substitute the same number in both expressions in Step 1

put the question -3/5+1/3 in simplest form

Answers

Answer:

-4/15

Step-by-step explanation:

-3/5+1/3 = -4/15

Andy £9500 Bevan £25000 Cheryl £13250 Deva £75000 Elliott £12750 Frankie £29500 Grace £11000 a) Work out the mean income to 2 DP. b) Deva was given a large bonus for 30 years service. Excluding Deva's income, what was the mean income for the remaining 6 workers to 2 DP?

Answers

Answer:

a. £25142.86

b. £16833.33

Step-by-step explanation:

According to the Question,

Given, The Salary Of 7 People in An Office are,

Andy ⇒ £9500

Bevan ⇒ £25000

Cheryl ⇒ £13250

Deva ⇒ £75000

Elliott ⇒ £12750  

Frankie ⇒ £29500

Grace ⇒ £11000

a. The mean income = Total Sum of Salaries ÷ Number of Workers

Mean = (9500+25000+13250+75000+12750+29500+11000) ÷ 7

Mean = 176000 / 7 ⇒ £25142.86

b. Now, The Mean income After Excluding Deva's Salary is,

Mean = (9500+25000+13250+12750+29500+11000) ÷ 6

Mean = 101000 / 6 ⇒ £16833.33

Grant is working at a veterinarian's office, and he needs to give a dose of medicine to a
25 kg dog. If a 10 kg dog needs 18 mL of the same medicine, how many milliliters of
medicine does the 25 kg dog need?

Answers

Answer:

45ml

Step-by-step explanation:

10 kg needs 18mL

So, 1kg = 1.8ml

25kg needs = 1.8kg/ml x 25 kg

25 kg dog would need 45ml.

Answer:

45 ml

Step-by-step explanation:

Weight of the dog and quantity of medicine is in Direct proportion.

Let the quantity required for 25 Kg dog =x

10 : 18 :: 25 : x

Product of extremes = product of means

10 * x = 18 * 25

[tex]x = \frac{18 * 25}{10}\\\\x = 9 * 5 \\\\x = 45[/tex]

A circular track has a radius of 70cm. If an athlete runs around it twice, how far has he run?​

Answers

Answer:140 cm

Step-by-step explanation: If they ran twice: 70*2=140cm

The runner has ran 280m because we need to know the whole length and the radius is 70 so the diameter is 140 times 2 is 280

Which expression can go in the blank to make the equation true?
-2.9 + 2.8 + ____________ = 0

Answers

Answer:

+0.1

Step-by-step explanation:

when we subtract 2.9 and 2.8 we get -0.1 and to make it zero we have to keep +0.1

When u subtract the to u get -0.1 so u got to add the the zero to get the positive 0.1

Please help asap thanks

Answers

Shahs. Dickens icjcns a icjcns icjcns. Did d

Math help please urgent please help

Answers

Answer:

Will cost 123 dollars.

Step-by-step explanation:

Find perimeter (50 m)

50 meters = 164 ft

164/10 ft = 16.4 ft

16.4 ft x 7.50 = 123 Dollars

Hope this helps. Consider giving brainliest :)

HELP PLS i don’t understand it so can someone please help me

Answers

Step-by-step explanation:

hey :) I can't help you BUT there is a very good app I recommend it's called "algebrator". I really hope it helps you :)

BRAINLIEST IF CORRECT You choose a movie at random from a list containing 8 comedy movies, 5 science fiction movies, and 7 adventure movies. What is the theoretical probability that the movie is not a comedy?

Select all that apply.

0.60

50%

252 fifths

353 fifths

60%

Answers

Answer:

0.60 & 60%

Step-by-step explanation:

Can someone explain the answer to 7.1x10^-3 ?

Answers

Answer:

[tex]{ \tt{7.1 \times {10}^{ - 3} }} \\ { \bf{negative \: means \: { \green{increase \: in \: decimal \: places}}}} \\ = { \tt{0.71 \times {10}^{ - 2} }} \\ = { \tt{0.071 \times {10}^{ - 1} }} \\ = { \tt{0.0071 \times {10}^{0} }} \\ { \boxed{ but \: {10}^{0} = 1}} \\ \therefore{ \tt{ = 0.0071 \times 1 = 0.0071}}[/tex]

Someone please help hurry I need to turn this in

Answers

Answer:

option a is right cz both have same value itz 1/root 2 hope it may help u

2. Alin ang mas mabigat, isang kilong pako o isang libong
gramo ng bigas?

Answers

Answer:

Isang Kilong Pako

Step-by-step explanation:

Ang pako po ay metal at ang bigas po ay grain lang kung ikukumpara mo sa isang butil ng bigas at pako, diba pako po ang mas mabigat, kung isang kilo pa po ehh mas mabigat na po iyon kaysa sa bigas.

Thank you Please mark as Brainliest :>

Really stuck, please help

Answers

Answer:

Step-by-step explanation:

what do you know about the angles of a triangle? They make 180° right?

since you know that, now you can find  angle  Y  , by using the triangle on the left,   and you can find X by using the triangle on the right,  then,  when you know  X & Y , use both of those to find  Z  :)   got it?

Answer:

we know that a triangle's angles MUST add up to 180 degrees.

We can look at the triangle with the angles of 55, 80, and y. As the angles must add to 180, 55+80+y must be 180. Therefore, we find that y is equal to 45.

We can look at the triangle with the angles of 100, 60, and x. As the angles must add to 180, 100+60+x must be 180. Therefore, we find that x is equal to 20.

We then look at the triangle formed from x, y, and z. As the angles must add to 180, x+y+z must be 180. x is 20, y is 45, so 20+45+z = 180. Thus, z is 115.

So:

y = 45

x = 20

z = 115.

ABC is a triangle right angled at A and D is a point on BC such that AD Ʇ BC. Show that AD2 = BD x DC. please answer fast URGENT

Answers

Answer:

The answer is below

Step-by-step explanation:

Pythagoras theorem states that for a right angled triangle, the square of the hypotenuse side is equal to the sum of the square of the remaining sides. The hypotenuse is the longest side (that is side opposite to the 90° angle).

In right angle triangle ABD:

AB² = AD² + BD²     (1)

In right angle triangle ACD:

AC² = AD² + CD²      (2)

Also:

AC² + AB² = BC²       (3)

But BC = BD + CD

AC² + AB² = (BD + CD)²     (4)

Adding equation 1 and 2 gives:

AB² + AC² = (AD² + BD²) + (AD² + CD²)  

AB² + AC² = 2AD² + BD² + CD²

substituting AC² + AB² = (BD + CD)²:

(BD + CD)² = 2AD² + BD² + CD²

BD² + 2(BD)(CD)+ CD² = 2AD² + BD² + CD²

2AD² = 2(BD)(CD)

AD² = BD * CD

Could someone please help me solve these?

Answers

In advance, I hope nothing I say sounds condescending; I just don’t know how much you know lol. Also geometry sucks

They’re vertical angles (they’re opposite angles when 2 lines intersect). Since vertical angles are congruent, we can set up an equation to solve for the value of x.
If they’re both equal, we’ll say exactly that in our equation!

2x + 90 = 4x + 30

Combine like terms

2x = 60

Simplify

x = 30

Now, just plug in 30 for one of the x values in either of the equations

2(30) + 90 = 150

Angle = 150 °

Hope this helped!
Other Questions
Which statement about printing is false?A. It helped revive interest in classical Roman and Greek art.B. It made books accessible to people and motivated them to get an education.C. It increased the value of original works of art. This animal has a protostome developmental pattern and a trochophore larva. Identify the major clades that this animal belongs to Write a report of 6 sentences about how can humans protect the environment from pollution This is the decision in a civil case regarding what will be made up to the injuredparty. what is it called Solve the equation. 5 = g/8 what does g equal Pls help i'm struggling with this 12 ducks fly overhead. Each of 6 hunters picks one duck at random to aim at and kills it with probability 0.6. What's the expected number of hunters who hit the duck they aim at? HELP DUE TODAY!!!!!! What aspects of Greek culture were adopted by the Romans to create the unique Greco-Roman culture they spread throughout the world? help me solve this pleaseeee Responsible behavior should be based on _______ and motivated by a will to _________ injury to self and others.feelings, causerules of safety, preventinstinct, inflict Fill in the blanksA dash is a human-readable description in the source code of a computer programPython has dash standard data typesA dash contains items separated by commas and enclosed within square bracketsA dash consists of key-value pairsdash cannot be changed and use parentheses what are the three strategies that the community could place to work with governmental structures to stop illegal dumping Dairy Queen lost $120 a month in January, February and March, because of the winter weather. What will be Dairy Queen's total loss after those 3 months?Select the choice below that best represents this situation. Group of answer choices1203=403120=3601203=3603120=360 you have 3 glasses that contain 8 oz each glass a glass A, your resident drinks only 3/4ths of it how much was consumed? how much was left? In the diagram, the measure of angle 3 is 105.A transversal intersects 2 lines to form 8 angles. Clockwise from the top, the angles are 1, 2, 3, 4; 5, 6, 7, 8.Which angle must also measure 105?Angle1Angle4Angle6Angle8 Help help help help help please ! Find surface area plzzzzz The owner of a small restaurant bought 75 kilograms of rice. Each week, the restaurant uses 4.5 kilograms of rice.Function r gives the remaining amount of rice, in kilograms, as a function of the number of weeks since the restaurantowner bought the rice.Complete the table. Type the answers in the boxes below.WEEKSKILOGRAMS OF RICE LEFT612 A coastal community is drafting a plan in case of a hurricane. Which action is an appropriate element for an effective plan? creating a map of areas that are not affected by storm surgeplotting the location of all fire hydrants along major travel routesrelocating any individuals who might be affected by the disasteridentifying multiple evacuation routes for each area of the community What is the difference between total output and labour productivity?